Evaluate x4 + 4x3 - 2x2 + 11x - 6 for x = 3

Answers

Answer 1

Step-by-step explanation:

[tex]f(x) = {x}^{4} + 4 {x}^{3} - 2 {x}^{2} + 11x - 6[/tex]

[tex]f(3) = {3}^{4} + 4. {3}^{3} - 2. {3}^{2} + 11.3 - 6[/tex]

[tex]f(3) = 81 + 4.27 - 2.9 + 33 - 6[/tex]

[tex]f(3) = 81 + 108 - 18 + 33 - 6[/tex]

[tex]f(3) = 198[/tex]

Answer 2

Answer:

198

Step-by-step explanation:

x4 + 4x3 - 2x2 + 11x - 6 for x = 3

3^4 + 4 * 3^3 - 2 * 3^2 + 11 * 3 - 6 =

81 + 4 * 27 - 2 * 9 + 11 * 3 - 6 =

81 + 108 - 18 + 33 - 6 =

198


Related Questions

A population of amoebas in a petri dish will triple in size every hour. At the start of an experiment the population is 800. The function y equals 800 times 3 to the power of x , where x is the number of hours, models the population growth. How many amoebas are in the petri dish after 9 hours?

Answers

Answer:

15,746,400

Step-by-step explanation:

This is an example of an exponential function.

1. The function is: [tex]800 * 3^{x}[/tex]

2. We can plug the number of hours into the equation.

3. population = 800 * [tex]3^{9}[/tex]

4. population = 15,746,400

Find the probability. Leave your answer in simplest fraction form.
You roll a six-sided die. The die shows an even number or a number
greater than three.

Answers

Answer:

5/6

Step-by-step explanation:

The numbers on a six-sided die are as shown:

1, 2, 3, 4, 5, 6

The even numbers are:

2, 4, 6

The numbers greater than 3 are:

3, 4, 5, 6

Both lists together are:

2, 3, 4, 5, 6

Because 5 out of 6 numbers satisfy these conditions, the probability of satisfying these conditions is 5/6.

The answer to this is 5,6

Deepak wrote out the steps to his solution of the equation startfraction 5 over 2 minus 3 x minus 5 plus 4 x equals negative startfraction 7 over 4 endfraction – 3x – 5 4x = –.

Answers

The solution is x=3/4

How can we solve given equation?

First, we will solve like terms. Then shift constant to other side and keep x on the same side to get the value of x.

We can solve given equation as shown below:

5/2-3x-5+4x=-7/4

(5-10)/2+x=-7/4

-5/2+x=-7/4

x=5/2-7/4

x= (10-7)/4

x=3/4

Hence, the solution is x=3/4.

Learn more about Linear equations here:

https://brainly.com/question/43297

#SPJ4

give the meaning of each expression.
16^5
y^4
7a^3

Answers

Answer:

16 to the power of 5

y to the power of 4

(7a) to the power of 3 or 7 times a to the power of 3

There are two numbers. One number is twice the other number. The difference of the smaller number and half the larger number is 20.

An equation created to find the smaller number will have

Answers

Step-by-step explanation:

There cannot be such a question because the difference of half of the big number and the small number will automatically be zero, but we can find it with the equation I circled on the paper I gave you. achievements

A vector has a magnitude of 8 in the 95 degree direction. what are the horizontal and vertical components?

Answers

Vertical component = 8 x sin(95) = 7.970 (4sf)
Horizontal component = 8 x cos(95) = -0.6972 (4sf)

Which function has zeros at x = -2 and x = 5?
O f(x) = x2 + 2x - 10
• f(x) = x2 - 2x - 10
O f(x) = X2 + 3x - 10
® f(x) = x2 - 3x - 10

Answers

Answer:

f(x)=x²-3x-10

Step-by-step explanation:

[tex]f(x) = x {}^{2} - 3x - 10 \\ to \: find \: x \: intercept \:o r \: zero \: substitute \: f(x) = 0\: \\ 0 = x {}^{2} - 3x - 10 \\ x {}^{2} - 3x - 10 = 0 \\ x {}^{2} + 2x - 5x - 10 = 0 \\ x(x + 2) - 5x - 10 = 0 \\ x(x + 2) - 5(x + 2) = 0 \\ (x + 2).(x - 5) = 0 \\ x + 2 = 0 \\ x - 5 = 0 \\ x = - 2 \\ x = 5[/tex]

therefore the zeros of the equation are x₁=-2,x₂=5

how many real solutions does this system of equations have?
y=x2+1
y=x

A. 1
B. 2
C. 3
D. 0

Answers

The system of equations has (d) 0 real solutions

How to determine the number of real solutions?

The system of equations is given as:

y = x^2+1

y=x

Substitute y=x in y = x^2+1

x = x^2+1

This gives

x^2 - x + 1 = 0

Calculate the discriminant using:

d = b^2 - 4ac

So, we have:

d = (-1)^2 - 4 * 1 * 1

Evaluate

d = -3

Because the discriminant is negative, the equation has no real solution

Hence, the system of equations has (d) 0 real solutions

Read more about system of equations at:

https://brainly.com/question/12895249

#SPJ1

Marta believes that the equation of the line of best fit for the scatterplot below is -
. Which statement best summarizes why Marta is likely incorrect?

Answers

Marta’s equation has a positive y-intercept, but the scatterplot shows a negative correlation.

What is a scatter plot?

The scatter plot is a manner in which data is presented as dots on a cartesian axes, The line of best fit is a description of the data that is presented in the scatter plot.

Hence, Marta is incorrect because Marta’s equation has a positive y-intercept, but the scatterplot shows a negative correlation.

Missing parts;

Marta believes that the equation of the line of best fit for the scatterplot below is y=-5/9x+23/9. Which statement best summarizes why Marta is likely incorrect?

Marta’s equation has a positive y-intercept, but the scatterplot suggests a negative y-intercept.

Marta’s equation has a positive y-intercept, but the scatterplot shows a negative correlation.

Marta’s equation has a negative slope, but the scatterplot suggests a negative y-intercept.

Marta’s equation has a negative slope, but the scatterplot shows a positive correlation.

Learn more about scatter plot:https://brainly.com/question/13984412

#SPJ1

Using mod, find the remainder of 3^51 when divided by 7. Please show steps on how to use modulu, am a bit confused.

Answers

Answer:

[tex]6[/tex]

Step-by-step explanation:

The gist of modular arithmetic in a nutshell: the numbers [tex]a[/tex] and [tex]b[/tex] are considered to be congruent by their modulus [tex]m[/tex] if [tex]m[/tex] is a divisor of their difference.

In mathematics: [tex]a \equiv_{m} b \Leftrightarrow (a - b) \vdots m[/tex]

Exemplifying this: [tex]6 \equiv_{7} -1[/tex] because [tex]6 - (-1) = 6 + 1 = 7[/tex], [tex]7 \vdots 7[/tex].

Let us have following equivalences: [tex]a \equiv_{m} b[/tex] and [tex]c \equiv_{m} d[/tex], then:  [tex](a - b) \vdots m[/tex] and [tex](c - d) \vdots m[/tex] by definition.

Properties:

1. [tex]a + c \equiv_{m} b + d \Leftrightarrow ((a + c) - (b + d)) \vdots m \Leftrightarrow (a + c - b - d) \vdots m \Leftrightarrow ((a - b) + (c - d)) \vdots m[/tex].

2. [tex]a - c \equiv_{m} b - d \Leftrightarrow ((a - c) - (b - d)) \vdots m \Leftrightarrow (a - c - b + d) \vdots m \Leftrightarrow ((a - b) - (c - d)) \vdots m[/tex].

3. [tex]ac \equiv_{m} bd \Leftrightarrow (ac - bd) \vdots m \Leftrightarrow (ac - bc - bd + bc) \vdots m \Leftrightarrow (c(a - b) + b(c - d)) \vdots m[/tex].

4. What if we have [tex]a \equiv_{m} b[/tex] twice? If we abide by property 3, we can come to the conclusion that [tex]a^2 \equiv_m b^2[/tex]. It is fair enough that there is room for the equivalence [tex]a^n \equiv_{m} b^n[/tex].

[tex]3^{51} = (3^3)^\frac{51}{3} = 27^{17} \equiv_{7} (-1)^{17} \equiv_{7} -1 \equiv_{7} 6[/tex].

We used property 4.

Keep in mind that any remainder cannot be a negative number.

Therefore, the remainder equals [tex]6[/tex].

What is the area of the given figure?
10 in.
O 192 units²
O
92.9 units²
O 101.8 units²
O167.2 units2
O
176.7 units2
6 in.
10 in.
9 in.

Answers

[tex]\huge\boxed{192\ \text{in}^2}[/tex]

There are three parts to this figure: a rectangle and two triangles that are congruent.

We'll add together the area for each to get the total area.

We'll start by finding the area of the rectangle. We don't know its length, so we need to find the bases of the triangles and add them together.

We know that [tex]a^2+b^2=c^2[/tex]. Substitute and solve for [tex]a[/tex]:

[tex]\begin{aligned}a^2+6^2&=10^2\\a^2+36&=100\\a^2+36-36&=100-36\\a^2&=64\\\sqrt{a^2}&=\sqrt{64}\\a&=8\end{aligned}[/tex]

Now, double this to get the total length of the rectangle, which is [tex]16[/tex] inches.

The area of the rectangle is equal to its length times its height:

[tex]16\cdot9=\underline{144}[/tex]

Now, we'll find the area of one of the triangles and double it since they're congruent.

The area of a triangle is one-half of its base times its height, which we then double.

[tex]2\left(\frac{1}{2}\cdot b\cdot h\right)[/tex]

The [tex]2[/tex] and the [tex]\frac{1}{2}[/tex] cancel each other out.

[tex]b\cdot h[/tex]

Substitute and solve:

[tex]8\cdot6=\underline{48}[/tex]

Finally, add the rectangle's area to the two triangles' area.

[tex]144+48=\boxed{192}[/tex]

Substracting 3x2+4x from 7x2+x+9 results in a polynomial. After subtracting 4x2-3x from this polynomial, the difference is ?

Answers

Answer:

9

Step-by-step explanation:

So first step is to subtract [tex]3x^2+4x[/tex] from [tex]7x^2+x+9[/tex]. In setting this up you get the following expression

[tex](7x^2+x+9)-(3x^2+4x)[/tex]

Distribute the negative

[tex]7x^2+x+9-3x^2-4x[/tex]

Group like terms

[tex](7x^2-3x^2)+(x-4x)+9[/tex]

Simplify:

[tex]4x^2-3x+9[/tex]

Now subtract [tex]4x^2-3x[/tex]. In setting this up you get the following expression

[tex](4x^2-3x+9)-(4x^2-3x)[/tex]

Distribute the negative:

[tex]4x^2-3x+9-4x^2+3x[/tex]

Group like terms

[tex](4x^2-4x^2) + (-3x+3x) + 9[/tex]

Simplify:

[tex]9[/tex]

seperate 90 into two parts so that one part is four times the other

Answers

Answer:

18 and 72

Step-by-step explanation:

the smaller part can be assigned as x while the larger will be 4x. Both numbers need to add up to 90, giving the equation: 4x+x = 90

Solve:

4x+x = 90

5x = 90

x = 18

4x = 72

A study on the latest fad diet claimed that the amounts of weight lost by all people on this diet had a mean of 23.4 pounds and a standard deviation of 6.8 pounds.
Step 2 of 2 : If a sampling distribution is created using samples of the amounts of weight lost by 63 people on this diet, what would be the standard deviation of the sampling distribution of sample means? Round to two decimal places, if necessary.

Answers

Using the Central Limit Theorem, the standard deviation of the sampling distribution of sample means would be of 0.86.

What does the Central Limit Theorem state?

It states that the standard deviation of the sampling distribution of sample means is given by:

[tex]s = \frac{\sigma}{\sqrt{n}}[/tex]

In which:

[tex]\sigma[/tex] is the standard deviation of the population.n is the sample size.

The parameters for this problem are given as follows:

[tex]\sigma = 6.8, n = 63[/tex].

Hence:

[tex]s = \frac{\sigma}{\sqrt{n}}[/tex]

[tex]s = \frac{6.8}{\sqrt{63}}[/tex]

s = 0.86.

More can be learned about the Central Limit Theorem at https://brainly.com/question/16695444

#SPJ1

Which describes the inverse operations used after the distributive property?
addition then division
subtraction then division
multiplication then subtraction
division then addition

Answers

The inverse operations used after the distributive property is B. subtraction then division.

How to illustrate the information?

The equation given goes thus:

5(x + 6) = 50.

5x + 30 = 50

5x = 20.

x = 4

Therefore, the inverse operations used after the distributive property is subtraction then division.

Learn more about distributive property on:

brainly.com/question/1780105

#SPJ1

Solve the equation for x.
6-√4+3x = 2

Answers

I think the answered is ×=20

6-2+3x=2
4+3x=2
3x=2-4
3x=-2
X=-2/3

Please answer this question fast (Lines and angles ch...class9)

Answers

The angle of y in the triangle is 30 degrees.

How to find angles in a triangle?

∠O = 180 - 30 - 70 (angles in a triangle)

∠O = 180 - 100(angles in a triangle)

∠O = 80°

Using vertically opposite angle principle and sum of angle in a triangle,

80 + y + 70 = 180

180 - 150 = y

y = 30 degrees

learn more on angles here: https://brainly.com/question/17738167

#SPJ1

If an artist combines 6 L of 33% by volume solution and 4 L of 60% by volume solution, what's the percent by volume of the resulting solution

Answers

The percent by volume of the resulting solution is 10L by 93 percent

How to determine the volume

The resulting solution = sum of the volumes of both solutions

Resulting solution = 6L × 33 percent + 4L × 60 percent

Resulting solution = 10 L × 93 percent

This is so because both solutions add up to form the resulting solution

Thus, the percent by volume of the resulting solution is 10L by 93 percent

Learn more about equivalent ratio here:

https://brainly.com/question/2328454

#SPJ1

aimee has 20 apples and wants to have the same amount of apples in each bag but ends up with more than one bag of apples and each bag to contain more than one apple
work out all the possibilities for all of the number of apples that she could use for each bag

Answers

Answer:

There are 4 possibilities: 2, 4, 5 or 10 apples in each bag.

===========

Let the number of apples in each bag is x, then the number of bags is 20/x.

Since the number of apples in each bag is greater than one and number of bags is greater than one, we have the following conditions:

x > 1 , 20/x > 1 ⇒ x < 20,x is factor of 20.

The factors of 20 between 1 and 20 are:

2, 4, 5, 10

The possibilities are:

If x = 2, then the number of bags is 20/2 = 10;If x = 4, then the number of bags is 20/4 = 5;If x = 5, then the number of bags is 20/5 = 4;If x = 10, then the number of bags is 20/10 = 2.

3 Quick algebra 1 Questions for 50 points!

For (lovetthannah9) or anyone who knows the answer! :)

Answers

Answer:

Step-by-step explanation:

6. The rate of change is essentially the slope. But in other words, it's how much the y changes as x increases by 1. If this rate of change is 0, that means the y-value is constant, while the x can be any real number. It can be generally given in the formula: y=b, which comes from simplifying the slope-intercept form: y=(0)x+b = b. You can also derive the same thing for a point-slope form: y-a = 0(x-b)=0 so y=a. In both cases, y is equal to some constant value. So graphing this makes a horizontal line.

7. To define a parallel line, you need the same slope, and a different y-intercept, which is under the assumption that the line has a slope that is definable. If the slope is definable, you'll have the equation: y=mx+b, where m is the same as the other line. Then you plug in the known values as x and y, to solve for b, which is the y-intercept. Now if the line is a vertical line, a parallel line can be defined as: x=a, where a doesn't equal the constant, the other vertical line is equal to. To make it pass through some point (c, d). Then you would simply set x=c, since the x is the only thing that matters, since the y-value is all real numbers, so it will eventually have the y-value d somewhere on the line.

8. To define a perpendicular line, you get the reciprocal of the slope and change the sign. So if the slope is: [tex]\frac{a}{b}[/tex] then it becomes [tex]-\frac{b}{a}[/tex] and vice versa depending on the sign. In this example, let's just say m=a/b, then the perpendicular slope would be -b/a. You can then plug this into the slope-intercept formula: [tex]y=-\frac{b}{a}+c[/tex] where c is the y-intercept. Then you plug in the known point as (x, y) and solve for c, the y-intercept. Now let's say for example that you have a vertical line: x=a. In this case you can see the slope as: [tex]\frac{1}{0}[/tex][tex]\frac{y_2-y_1}{a-a}[/tex]. Of course this isn't definable, but if you take the reciprocal you get something that is: [tex]-\frac{0}{y_2-y_1}[/tex] which will always evaluate to 0. This means you get a horizontal line, since the slope is 0. This means if you have a vertical line, any horizontal line should be perpendicular, which makes sense, since it should form a 90 degree angle when they intersect, because they're straight lines. To make sure that horizontal line passes through the point (b, c), you simply set the y equal to c. So y=c, will pass through (b, c), since the c is constant, and the x can equal anything so somewhere on the line it will intersect (b, c). But let's say we had a horizontal line, the reciprocal can be defined as: [tex]\frac{a-a}{x_2-x_1}[/tex] where a is the y constant. If you take the reciprocal, then you have a-a in the denominator, which gives you an undefined slope, because the perpendicular line to a horizontal line, is a vertical line. To ensure this perpendicular line passes through the point (b, c). You simply set x equal to b. so that x=b.

#6

y=mx+c

if m is 0

y=c is the Equation

Hence line is parallel to x axis

#7

parallel lines have equal slopes

so we can use the point slope form of line to find the equation

y-y_1=m(x-x_1)

#2

Same process like no 7 but

slope of perpendicular line is negative reciprocal of given line's slope

Please help! Consider the sequence {20, 17, 14, 11, 8, 5, 2...}.

Answers

Answer:

c.) 38

Step-by-step explanation:

[tex]\sum\limits_{n=3}^6 a_n[/tex]   means "summation of the all the elements starting from the 3rd element to the 6th element".

The 3rd element in the series is 14, and the 6th element is 5; we have to add these and all the elements between them together.

∴    [tex]\sum\limits_{n=3}^6 a_n[/tex]    =  14 + 11 + 8 + 5

                   = 38

Using the following image, solve for SR. Look at the image closely.
What would SR be? Giving Brainly!

Answers

Step-by-step explanation:

I assume

SR = 2x + 23

RQ = x + 21

if that is true, then the situation is completely simple :

14 = (2x + 23) + (x + 21) = 3x + 44

3x = -30

x = -10

SR = 2×-10 + 23 = -20 + 23 = 3

RQ = -10 + 21 = 11

[tex]\quad \huge \quad \quad \boxed{ \tt \:Answer }[/tex]

[tex]\qquad \tt \rightarrow \:x = -10 [/tex]

____________________________________

[tex] \large \tt Solution \: : [/tex]

[tex] \qquad \tt \rightarrow \: SR + RQ = SQ[/tex]

[tex]\qquad \tt \rightarrow \: 2x + 23 + x + 21 = 14[/tex]

[tex]\qquad \tt \rightarrow \: 3x + 44 = 14[/tex]

[tex]\qquad \tt \rightarrow \: 3x = 14 - 44[/tex]

[tex]\qquad \tt \rightarrow \: 3x = - 30[/tex]

[tex]\qquad \tt \rightarrow \: x = - 10[/tex]

Now,

[tex] \qquad \tt \rightarrow \: SR = 2x + 23 [/tex]

[tex] \qquad \tt \rightarrow \: SR = 2(-10) + 23 [/tex]

[tex] \qquad \tt \rightarrow \: SR =-20+ 23 [/tex]

[tex] \qquad \tt \rightarrow \: SR = 3 \:\: units [/tex]

Answered by : ❝ AǫᴜᴀWɪᴢ ❞

X = square root=
Help me please thanks

Answers

The value of x will be equal to 33.94 units.

What is trigonometry?

Trigonometry is the branch of mathematics that set up a relationship between the sides and angle of the right-angle triangles.

Given that:-

Given that the radius of the circle is 12 units which are PO = 12 and OR = 12, QR = x , ∠O = 60 So ∠Q = 30.

First, we will calculate the Length OQ by angle property in triangle OQR.

[tex]Sin 30=\dfrac{Perpendicular}{Hypotenuse}[/tex]

[tex]Sin 30 = \dfrac{12}{OQ-12}[/tex]

[tex]\dfrac{1}{2}=\dfrac{12}{OQ-12}[/tex]

OQ - 12 = 24

OQ = 36

Now applying the Pythagorean theorem in the triangle OQR.

H² = P² + B²

36² = 12² + x²

x² = 1296 - 144

x = √1152

x = 33.94

Therefore the value of  x will be equal to 33.94 units.

To know more about Trigonometry follow

https://brainly.com/question/24349828

#SPJ1

Determine if each table below represents a linear function, quadratic function, or neither.

Answers

The table below is a quadratic function.

What is a quadratic function?

A quadratic function is an Algebraic function with the power of its variable as 2.

This function normally has three algebraic terms, the x-square term, the x-term and the constant term.

Analysis:

The X-column values increases from -2 to 1, while the f(x) column which is the y-column increase from -9, until it gets to 1, where the value falls to -5.

A typical behavior of an inverted-v curve which is a quadratic curve.

If it were a linear function, as x values increase, y value would either keep increasing or decreasing, it does not change its orientation.

Learn more about quadratic curves: brainly.com/question/1214333

#SPJ1

Which statement regarding the diagram is true W, X,Z,Y

Answers

The statement that's true about the triangle is that WXY + YXZ = 180°.

How to illustrate the information?

It should be noted that in the question, the options are related to a linear pair.

According to the linear pair, when a line cuts another line at a point, then the sum of the adjacent angles formed at the point will be 180°.

Therefore, WXY + YXZ = 180°.

The correct option is C.

The complete question is:

Which statement regarding the diagram is true?

m∠WXY = m∠YXZ

m∠WXY < m∠YZX

m∠WXY + m∠YXZ = 180°

m∠WXY + m∠XYZ = 180°

Learn more about triangles on:

brainly.com/question/4001545

#SPJ1

For the following exercises, solve each inequality and write the solution in interval notation.
31. | 3x − 4 | ≤ 8

Answers

Answer:

The solution set in interval form is [tex]$\left[\frac{-4}{3}, 4\right]$[/tex].

Step-by-step explanation:

It is given in the question an inequality as [tex]$|3 x-4| \leq 8$[/tex].

It is required to determine the solution of the inequality.

To determine the solution of the inequality, solve the inequality [tex]$3 x-4 \leq 8$[/tex] and, [tex]$-8 \leq 3 x-4$[/tex].

Step 1 of 2

Solve the inequality [tex]$3 x-4 \leq 8$[/tex]

[tex]$$\begin{aligned}&3 x-4 \leq 8 \\&3 x-4+4 \leq 8+4 \\&3 x \leq 12 \\&x \leq 4\end{aligned}$$[/tex]

Solve the inequality [tex]$-8 \leq 3 x-4$[/tex].

[tex]$$\begin{aligned}&-8+4 \leq 3 x-4+4 \\&-4 \leq 3 x \\&-\frac{4}{3} \leq x \\&x \geq-\frac{4}{3}\end{aligned}$$[/tex]

Step 2 of 2

The common solution from the above two solutions is x less than 4 and [tex]$x \geq-\frac{4}{3}$[/tex]. The solution set in terms of interval is [tex]$\left[\frac{-4}{3}, 4\right]$[/tex].

rewrite 4 1/2=2
please

Answers

The logarithmic expression of 4^(1/2) = 2 is [tex]\log_4(2) = \frac 12[/tex]

How to rewrite the expression?

The expression is given as:

4^(1/2) = 2

Take the logarithm of both sides

log(4^(1/2)) = log(2)

Apply the change of base rule

1/2log(4) = log(2)

Divide both sides by log(4)

1/2 = log(2)/log(4)

Change the base

[tex]\frac 12 =\log_4(2)[/tex]

Rewrite as:

[tex]\log_4(2) = \frac 12[/tex]

Hence, the logarithmic expression of 4^(1/2) = 2 is [tex]\log_4(2) = \frac 12[/tex]

Read more about logarithmic expression at:

https://brainly.com/question/24211708

#SPJ1

URGENT!!!WORTH 27 POINTS!!!

Doug can download new songs for $1.19 each. Write an equation to show how many songs he can download for $12.00.

A. 12x=1.19
B. 12+x=1.19
C. 1.19+x=12
D. 1.19x=12

Answers

Answer:

1.19x = 12

Step-by-step explanation:

To write an equation, we multiply the cost of each song by the number of songs, x

1.19 * x

This is equal to the total amount he is allowed to spend

1.19x = 12

Answer:

D

Step-by-step explanation:

If Doug can download new songs for $1.19 each, then he will be able to download 12 songs.

For which value of k is the value of k(k-2)(k+1) negative

Answers

Answer:

[tex](-\infty, -1)\ \cup\ (0, 2)[/tex]

Step-by-step explanation:

So if you expand out the two binomials (k-2)(k+1), you'll get: [tex]k^2+k-2k-2[/tex]. which simplifies to: [tex]k^2-k-2[/tex]. Multiplying this by the k gives you: [tex]k^3-k^2-2k[/tex]. As you can see the degree is odd, this means that this polynomial will have two opposite end behaviors. And as you can see the leading coefficient is positive, meaning that this function will go towards positive infinity as k goes towards positive infinity. Also we if look at the original equation given, it's in factored form, with the zeroes as k=0, k=2, and k=-1. So given this we can draw a simple graph to see when the value of the equation is negative. If you look at the graph I drew you'll see that it's negative from (-infinity, -1) and then negative from (0, 2)

Which ordered pairs make the equation true? 3x+2y=−7 Select each correct answer. (3, −8)
(−3, 1)
(−2, −1)
(1, −4)

Answers

SOLVING

[tex]\Large\maltese\underline{\textsf{A. What is Asked}}[/tex]

Which ordered pairs make the equation [tex]\bf{3x+2y=-7}[/tex] true? Select all that apply. 3 options are given

[tex]\Large\maltese\underline{\textsf{B. This problem has been solved!}}[/tex]

We can tell which ordered pairs make equations true by plugging in the coordinates.

[tex]\mathbb{ORDERED\;PAIR\;NUMBER\;ONE}[/tex]

[tex]\bf{3(3)+2(-8)=-7}[/tex] | simplify

[tex]\bf{9-16=-7}[/tex] |simplify

[tex]\bf{-7\equiv-7}[/tex] | this one checks

[tex]\mathbb{ORDERED\;PAIR\;NUMBER\;TWO}[/tex]

[tex]\bf{3(-2)+2(-1)=-7}[/tex] | simplify

[tex]\bf{-6-2=-7}[/tex] | simplify

[tex]\bf{-8\neq-7}[/tex] | this one does not check

[tex]\mathbb{ORDERED\;PAIR\;NUMBER\;THREE}[/tex]

[tex]\bf{3(1)+2(-4)=-7}[/tex] | simplify

[tex]\bf{3-8=-7}[/tex] | simplify

[tex]\bf{-5\ne-7}[/tex] | this one does not check

[tex]\rule{300}{1.7}[/tex]

[tex]\bf{Result:}[/tex]

              [tex]\bf{=The\;First\;Option[/tex]

[tex]\boxed{\bf{aesthetic \not101}}[/tex]

Other Questions
Select the correct answer from each drop-down menu. Which system of inequalities does the graph represent? Which test point satisfies both of the inequalities in that system? The graph represents the system of inequalities . The test point satisfies both of the inequalities in the system represented by the graph. In the russian federation, which official holds the most power? a. the duma b. the president c. the prime minister d. the monarch please select the best answer from the choices provided a b c d What is the nightingale shady wood to the skylark African fish eagles are large raptors that have a wide range across sub-Saharan Africa. Which of these organisms do you think would have a niche similar to that of an African fish eagle?A- American bald eagle B- European blue jayC- White-tailed deerD- Mountain lion Whats the length of three pipes, when combined they equal 72m, how do I find the answer 1. Describe theoverall changes inmatter that occurduring the light-independentreactions. What doesthe plant start withand what does it endwith? What is thesource of energy thatpowers this change? How did the selective service act result in the AEF arriving in France Which of the following describes a difference between NAFTA and the Kyoto Protocol? A. NAFTA deals with trade policy, while the Kyoto Protocol deals with the environment. B. NAFTA is a global organization, while the Kyoto Protocol includes only Asian states. C. NAFTA allows members to pass tariffs, while the Kyoto Protocol promotes free trade. D. NAFTA uses embargoes to further its political goals, while the Kyoto Protocol does not. If you work for a company and have a 401(k) retirement account, the account contains money you contributed to your retirement and money contributed by your employer. When you retire, which of the following will these payments be?A. Welfare paymentsB. PensionC. Transfer paymentsD. None of the above Find y. Help me please thank u:) The Serbian/Yugoslav leader who presided over the attempts to prevent Kosovo from seceding from Yugoslavia was: an airplane flies 3300 miles with the wind in 3 hours 18 minutes. it takes 4 hours and 24 mins to make the return flight against the same wind. find the speed of the airplane and the speed of the wind. a = 23 ft.;c = 4 ft.A = 83 sq. ft.TrueFalse Question 31 of 34Which situation shows a nonrenewable resource in use? Leonard has built a crate for his car. The crate measures (x+5) ft by (x+1) fr by (2x+3) ft. Write an expression that represents the volume in the crate in cubic feet The following formula gives the total surface area S of a cylinder with radius r and height h.S=2r(r+h)Find the total surface area of a cylinder with radius 4 and height 8. Enter an exact answer in terms of . Do not include S= in your answer. Over the past half century in the United States, ethnic and racial diversity have been ________. A newly discovered species of bird is found to only lay a single egg and both parents are necessary to raise the chick to adulthood. What would be the most successful mating strategy for this species 4 . A negative charge -0.450 exerts an upward 0.150N force on an unknown charge 0.250m directly below it . ( a ) what is the unknown charge ( magnitude and sign ) ? ( b ) what are the magnitude and direction of the force that the unknown charge exert charge exerts on the -0.450 charge ? The point-slope form of the equation of a line that passes through points (8, 4) and (0, 2) is y - 4 = 1/4 (x-8). WhatIS the slope-intercept form of the equation for this line?O y= 1/4x -12O y= 1/4x-4O y=1/4x+2O y= 1/4x+6